You are on page 1of 39

READING COMPREHENSION Concepts

Reading comprehension includes the ability to identify main ideas, locate specific details, understand passage
structure, and make inferences regarding a written work. A deep understanding of a passage requires the reader to
understand not only the meanings of the individual words that comprise that text, but also the way that each
sentence functions within the context of the whole piece. Aside from basic literacy, there are a number of logical
reasoning competencies that aid in deeper comprehension, particularly with regard to making inferences and
understanding passage structure.

Typically, the Verbal Section on the GMAT includes 4 Reading Comprehension passages, with 3 to 4 questions per
passage, for a total of 13 to 14 Reading Comprehension questions out of the 41 verbal questions. Passages will
contain 3 or 4 questions and are about 200 to 400 words long. Each passage engages with a specialized topic or
opinion in the Humanities, Social Sciences, Science, or Business, but no specific outside knowledge of the material is
required; all questions refer to what is stated or implied in the text. The writing in these passages is serious and
sophisticated.

The Challenge:

Think of the reading comprehension section as if it were a reality TV show where you are dropped in the middle of a
jungle with no clues about where you are or how to proceed. On the GMAT, a reading passage will be dropped in
front of you and you will have no background on it whatsoever. Imagine encountering an essay where:

1. You don't know what the title is.


2. You don't know who the author is.
3. You don't know when or where it was published.
4. You can't see the paragraphs before or after the essay.
5. You don't have enough time to fully read it.
6. The content is dense, boring, and academic, smeared with jargon, and covers a topic you have little
knowledge about.
7. And your mastery of those 400 words will determine your future business school and career.

Moral: You're going to need a compass. That is, a planned strategy and a lot of preparation coupled with tons of
practice.

Distinguishing between Fact and Opinion

Often, your ability to answer a reading comprehension question correctly will depend upon your ability to distinguish
between fact and opinion. You may need to determine whether an author thinks something is true or whether the
author knows something to be true to determine the main idea or draw logical conclusions about the text. First, here
is a quick review of definitions. A fact is something known for certain to have happened, to be true, or to exist. An
opinion, on the other hand, is something believed to have happened, to be true, or to exist. The key difference
between fact and opinion lies in the difference between believing and knowing. Opinions may be based on facts, but
they are still what people think and believe, not what they know. Opinions are debatable; facts are not. Two different
people would have a hard time debating a fact, but they could debate forever about which opinion is more valid. Note
that people can also debate about how to interpret facts, but they would have to agree on the facts themselves. A
good test for whether something is fact or opinion is to ask two questions:

Ivy-GMAT Exclusive GMAT-Prep for 99th percentile (760-800 range) aspirants not for everyone! Dont join us if you are
not willing to put in the requisite amount of work.
1. Can this statement be debated?
2. Is this something known to be true?

If you can answer yes to the first question, it is probably an opinion. If you can answer yes to the second question, it
is probably a fact.

In addition, consider the nature of the claim. If the statement is prescriptiveif it is describing what someone should
or ought to dothen the statement is an opinion, as in the following examples:

1. You should try advertising on the radio.


2. We ought to offer a better severance package.
3. I had better confirm this appointment before I book a flight.

Words that show judgment or evaluation, like good, bad, interesting, and important, usually also signal an opinion.

Consider this example:

Employee benefits should include coverage for alternative medicines such as acupuncture and massage therapy.

This statement is clearly debatable and could be argued either way. In an effective argument, this opinion would be
supported by and based upon facts. For example, if you had chronic back pain that was not alleviated by traditional
medical approaches but that disappeared after three weeks of acupuncture, you could use this fact to support your
opinion. In addition, you could cite the fact that the alleviation of pain saved your insurance company hundreds to
thousands of dollars in additional visits to back pain specialists and other medical practitioners. You might also cite
statistics, such as a recent survey that showed more than 60% of patients with chronic back pain reported relief after
one month of acupuncture. These facts, which are non-debatable, would support your opinion, making it more
reasonable and therefore more valid. It is easy to see how this information is relevant to the critical-reasoning
questions (which ask you to evaluate arguments) and the AWA questions (which ask you to write your own
argument). It is also relevant to reading comprehension questions because knowing the authors opinion and how the
author supports that opinion can help you draw appropriate conclusions from the text.

The finer aspects of the reading process: Comprehending a passage is an ebb and flow process. Analyzing a
passage to understand how it is constructed can be compared to dismantling an engine to understand how it was
builtyou may stop occasionally and reassemble parts of it to review what you just did; then proceed again to
dismantle more. Likewise, when reading a passage, you may first read and mentally annotate a paragraph
(disassembling it) and then go back and skim to reassemble it. During this process, comprehension proceeds from the
global to the specific.

Conjecture (Main Idea)



General Understanding (authors attitude, logical structure)

Specific Understanding (Logical Details)

Incorporation (What if?)

In the conjecture stage, we form a tentative main ideaone which we may have to modify or even reject as we read
more deeply into the passage. In the general understanding stage, we develop a feel for the authors tone and
discover the schema that s/he uses to present her ideas. In the specific understanding stage, we fill in the minor gaps
in our understanding. Finally, in the incorporation stage, we integrate the ideas presented in the passage into our own
thought process. We now understand the ideas sufficiently to defend them, apply them to other situations, or
evaluate their validity in a hypothetical situation. Only with complete understanding of the passage can this be done.

Ivy-GMAT Exclusive GMAT-Prep for 99th percentile (760-800 range) aspirants not for everyone! Dont join us if you are
not willing to put in the requisite amount of work.
Normally all RC passage will do one of the following:

Describe
Evaluate
Persuade / Suggest

Describe Evaluate Persuade

Essay arguing that the


Newspaper article on Book review of Margaret
Humanities romance is a distinctly
Harlem Renaissance Atwood novel
American literary genre
Newspaper opinion column Environmentalists op-ed piece
Textbook Section on Civil
Social Science about a politicians arguing for tighter controls on
War
economic plan the use of a new chemical
Commentary in a physics A scientists explanation of why
Museum pamphlet on
Science journal on a new plan for he believes the Big Bang
volcanoes
solar power Theory is wrong
Harvard Business Review A corporate managers letter to
Newspaper account of the
assessment of a new a newspaper defending his
Business collapse of the real estate
theory of corporate company against accusations
market
leadership of mismanagement

If youre like most GMAT test-takers, youll experience at least one of the following problems as you tackle Reading
Comprehension:

Your concentration is poorperhaps because of your lack of familiarity with or interest in the topic, or
perhaps due to general test anxiety.

Your reading pace is slowso you have trouble finishing the Verbal section in time.

To answer each question, you find yourself searching the passage again and again to find the information you
need.

You have trouble narrowing down the answer choices to one thats clearly the best, especially between the
last two.

Believe it or not, all of these problems are due to the same bad habit: passive reading, by which you simply read the
passage from start to finish, giving equal time and attention to every sentence without thought as to what particular
information might be key in answering the questions. You might call this approach the osmosis strategy, since
youre hoping to absorb what you need to know by simply allowing your eyes to glaze over the words.

Whats the likely result of this osmosis strategy? You might remember some scattered facts and ideas, which will help
you respond correctly to some easier questions. But the passive mind set wont take you very far when it comes to
most of the questions, which measure your ability to understand the ideas in the passage rather than to simply recall
information. Understanding a passage well enough to answer all the questions requires a highly active frame of
mindone in which you constantly interact with the text as you read.

Ivy-GMAT Exclusive GMAT-Prep for 99th percentile (760-800 range) aspirants not for everyone! Dont join us if you are
not willing to put in the requisite amount of work.
The Test-Makers Top 10 Wrong-Answer Ploys

1. The response distorts the information in the passage. It might understate, overstate, or twist the passages
information or the authors point in presenting that information.
2. The response uses information from the passage, but does not answer the question. The information cited
from the passage isnt useful to respond to the question at hand.
3. The response relies on speculation or an unsupported inference. It calls for some measure of speculation in
that the statement is not readily inferable from the information given.
4. The response is contrary to what the passage says. It contradicts the passages information or runs contrary
to what the passage infers.
5. The response gets something in the passage backwards. It reverses the logic of an idea in the passage,
confuses cause with effect, or otherwise turns information in the passage around.
6. The response confuses one opinion or position with another. It incorrectly represents the viewpoint of one
person (or group) as that of another.
7. The response is too narrow or specific. It focuses on particular information in the passage that is too specific
or narrowly focused in terms of the question posed.
8. The response is too broad (general). It embraces information or ideas that are too general or widely focused
in terms of the question posed.
9. The response relies on information that the passage does not mention. It brings in information not found
anywhere in the passage.
10. The response is utter nonsense. It makes almost no logical sense in the context of the question; its
essentially gibberish.

Heartstrings:

Beware of answer-choices that contain extreme emotions. Remember the passages are taken from academic journals.
In the rarefied air of academic circles, strong emotions are considered inappropriate and sophomoric. The writers
want to display opinions that are considered and reasonable, not spontaneous and off-the-wall. So if an authors tone
is negative, it may be disapprovingnot snide. Or if her tone is positive, it may be approvingnot ecstatic.

Furthermore, the answers must be indisputable. If the answers were subjective, then the writers of the GMAT would
be deluged with mails from angry test takers, complaining that their test-scores are unfair. To avoid such a difficult
position, the writers of the GMAT never allow the correct answer to be either controversial or questionable.

Keywords: Keywords tell readers what is about to happen next. Some signal words tell readers about the sequence;
others tell about a similarity or difference. Students often overlook these clues. Make sure you think aloud for
students to model how these words can be used.

Sequence Words: after, afterward, ahead of, all through, as before, beforehand, during, earlier than, first, second,
third finally, later, now, prior to, sooner than, subsequently, then, throughout, while

Restatement or Synonym Words


Also, as well as, by the same token, correspondingly, equally, equally so, especially, for example, in that, in the same
way, just as, likewise, similarly, such as these, too

Contrast or Antonym Words


Alternatively, although, apart from, but, by contrast, contrary to that, conversely, despite, even though, however, in
contrast, in spite of this, nevertheless, nonetheless, notwithstanding, on the other hand, regardless, some but
others, still, then again, yet

Ivy-GMAT Exclusive GMAT-Prep for 99th percentile (760-800 range) aspirants not for everyone! Dont join us if you are
not willing to put in the requisite amount of work.
Four basic tasks:

The passage below demonstrates four basic capacities that factor into the reading of a new text. After a complete
reading, try employing the following four skills:

Identify the main idea


Locate a specific detail
Consider the organization of the paragraphs
Make at least one inference

Many people argue that the rise of cable news has degraded national political discourse. They blame the
starkness of the left-right political divide on the sensationalist presentation style employed by 24-hour news
networks in their effort to make journalism more entertaining and palatable. As evidence of the trend,
opponents point to the dramatic increase in the number of news commentatorspartisan newscasters who
argue vociferously for one party or another. Such programs, critics say, contribute to the hostile, uninformed
political climate that can be held responsible for the lack of bipartisanship in government.

Ultimately, however, cable news has been vindicated by its opponents. Demographics show that its loudest
detractors come from demographics that have been historically apolitical, namely, young people, married
women, and lower-middle class workers. Since the launch of CNN in 1980, cable news viewership rates have
risen proportionally with greater political participation and activism by groups that were previously apolitical.
Studies of TV ratings trends and polling data show that college students and housewives are the largest
consumers of cable news. In other words, detractors of cable news blame the very source of their political
awakening for the frustrating political climate of which it has made them aware.

Main Idea: Though people blame cable news for the toxic political climate, the growth of its viewership has led to
greater political participation by the very classes that denounce it.

A Detail: CNN launched in 1980.

Passage Structure: The first paragraph explains that many people attribute a particular cause to a supposed problem;
this attribution is undermined in the second paragraph.

An Inference: Before 1980, housewives and college students were less politically active than they are today.

Ivy-GMAT Exclusive GMAT-Prep for 99th percentile (760-800 range) aspirants not for everyone! Dont join us if you are
not willing to put in the requisite amount of work.
Our RC blasphemies that WORK on the test day:

RC is the most crucial among all test areas: Our experience tells us that, on average, each RC question
weighs almost twice as much as each CR question and almost thrice as much as each SC question.

You cannot cross the 700 mark if your RC is bad: Our experience tells us that students really good at
Quant, SC, and CR, but average at RC tend to stop short of 700. We have seen countless 670-690 scorers,
who, only for want of a superlative performance on RC, could just not breast the coveted 700+ Club tape.

RC under normal concentration and relaxed pace can be very different from RC under trying exam
conditions: fourth hour on the test, race against the clock, the mind refusing to understanding any word at
all, and the imperative to finish the test ensure that, more often than not, the victim (and the biggest culprit)
is RC.

When you dont do well on RC, you lose 3-4 questions in a row: Consecutive errors are penalized so
severely on the test that, at any given point on the test, 3-4 consecutive errors can bring your current score
by almost 100 points. Bad RC ensures that you have at least 3-4 such chunks of score-drops. OUCH!!

Normal pace: Every conceivable resource possible on GMAT RC will ask you to speed-read, skip, skim, etc.
Trust us: nothing could be more gimmicky. Dont speed-read GMAT RCs. It may be curtains! Your natural
speed must improve after practicing 300+ passages.

Focus on understanding: Every conceivable resource possible on GMAT RC will ask you to understand only
the main idea and skip the details. We have even seen such advice: read only the first line of each para etc.
Trust us: nothing works on the test day. There is no substitute for a reasonable understanding. These
resources / books / instructors / test-prep companies take passages (that lend themselves to such gimmicks)
and show the application of the gimmicks that work on a particular passage.

Read the full passage first: Some books / test-prep companies will be asked to read the questions first.
Nothing could be a bigger disservice to oneself.

Dont skip: You cant afford to skip information. Your understanding of the passage will be distorted
otherwise.

Dont skim: Skimming is a self-contradictory term. Without reading everything, how can one decide the
grain from the chaff? Again, understand that there is no substitute for understanding.

Dont reread: Develop a habit of reading the passage only once and understanding it well. Most of us tend
to reread the first few lines. The signal to our subconscious brain is: let me not understand it fully. Anyway I
have to come back and reread. It may be tough to get rid of this habit but this is extremely crucial to train
your brain not to reread.

Dont write anything at all: The mother of all wrong pieces of advice on the GMAT is: write / make a
skeletal / make a structure / annotate etc. UGH!! This is the WORST advice one can give a student.

o Why should you not write?

 You are so pressed for time on the test day that you never use your notes. Why learn and
rely upon something that will not work on the test day?

 Invariably our experience says that students never use what they write. Why waste precious
time?

 Once your brain gets a subconscious signal that you can write, the understanding vanishes.
You are just taking notes passively. RC is about active application, not passive note-taking.

Must go back while solving the questions: A lot of times, students rely upon their memory and miss out
on the intentional distortion used so effectively by the test-makers (psychometricians). These people are
masters of making the wrong answers that seem more attractive than the right answers. The best way to
Ivy-GMAT Exclusive GMAT-Prep for 99th percentile (760-800 range) aspirants not for everyone! Dont join us if you are
not willing to put in the requisite amount of work.
avoid this is to go back to the passage to justify each word in the answer choices. Extreme WORD
JUSTIFICATION is the name of the game.

Dont do more general reading; solve more RC exercises: The ubiquitous advice that people get when
they do badly on RC is: READ, READ, and READ.

WRONG! If this were to be the case, you would be answering the questions right in the first place.
You have been reading every day for at least the last 10 years, at least.

Solve more and more RC passages RIGHT!

Dont bother about vocabulary: A lot of people bother about understanding each word in the passage to
an obsessional level. Indeed, a good vocabulary can help you but there is no point in developing superior
vocabulary NOW. The efforts can be completely disproportionate compared to the returns. So the best bet is
to guess the meaning of a new word by the context.

Plan four-hour nonstop sittings only with RC to make sure you concentrate well on the test. If
you genuinely want the 740+ score, you cant skip this piece of advice.

Dont solve anything else except what has officially appeared on Standardized American Tests such as the
GMAT, the GRE, or the LSAT. There are at least 500 genuine such passages available for you to practice with.

Beware of the central opinion / theme / main idea / tone etc. while reading the passage.

Ideally for a passage with 3 questions, you must not take more than 5 minutes and for a passage with 4
questions, you must not take more than 6.5 minutes.

Ivy-GMAT Exclusive GMAT-Prep for 99th percentile (760-800 range) aspirants not for everyone! Dont join us if you are
not willing to put in the requisite amount of work.
OCTAVE
A guaranteed formula for 100% success on GMAT RC
OCTAVE stands for:

Opinion: Always look for the personal opinion of the author or the main point made by the author. Other
peoples opinions are not so important.
Contrast Words: Watch out for contrast words: these are important because usually there is surely a general /
inference question from these locations. The common contrast words are:
Alternatively, although, apart from, but, by contrast, contrary to that, conversely, despite, even though,
however, in contrast, in spite of this, nevertheless, nonetheless, notwithstanding, on the other hand,
regardless, some but others, still, then again, yet

Tone: Always try to predict the tone of the passage as positive / negative / neutral. This is a big help.
Avoid superlatives / extreme words / strong words / emotional words in the option choices. These are almost always
wrong.
V erify each word: This is the heart of RC. You must make sure that each word written in the choices must be
justifiable from the passage.
Eliminate: Dont be fixated on an answer choice if it seems attractive. Follow elimination of wrong ones rather than
selection of the right one.

OCTAVE will be discussed over 100 passages in our classes.

Ivy-GMAT Exclusive GMAT-Prep for 99th percentile (760-800 range) aspirants not for everyone! Dont join us if you are
not willing to put in the requisite amount of work.
Further reading more on RC
Identifying main idea of the passage

All passages center around one main idea; the remainder of the text presents evidence that supports this primary
point. All paragraphs and statements within the passage take on specific roles with regard to advancing the main
idea.

Remember, the main idea of a passage must encompass the entire passage. If any major part of the passage does
not seem to relate to a given answer choice, that choice is most likely not the main idea. Consider the example below:

All available research points to the fact that music education can significantly bolster students' academic skills,
particularly when this education is imposed during early childhood. The most notable academic improvement
associated with music education, surprisingly, occurs in affected student's performance in mathematics. A
recent study, which compared students in school districts all around the world, found that students who
began receiving music education in kindergarten scored significantly higher on standardized math tests than
did those who began after third grade. Those students who began in third grade, in turn, scored significantly
higher than did those who received no music education at all. In fact, correlation studies attempting to link
several different factors to higher math test scores have found music education to be the most significant of
these factors.

What is the author's main point?


A. An important research study determined that students whose music education commenced in kindergarten fared
the best on standardized tests.
B. Music education has the potential to strengthen students' academic skills.
C. The study regarding the effect of music education on academic success was conducted in an objective and
scientific manner.
D. The overall effectiveness of music education in improving a child's academic performance in mathematics is
correlated with the age at which that child begins to receive instruction in music.
E. Schools are willing to employ strategies that have the potential of bolstering the academic performance of their
students.

Choice B is correct. The author believes that music education can improve students' academic skills; all other
statements in this passage serve as evidence to support this belief. Choices A and D are examples of this evidence,
while choices C and E represent unspoken assumptions that underlie the argument.

Inference questions

An inference is an idea that is not itself explicitly stated, but that can be logically concluded from what is explicitly
stated. An inference for test purposes refers to something that must be true based on the evidence provided.
Remember, inferences are not leaps in logic. In fact, they are quite the opposite: they are statements that are
logically implicated in the argument and should follow directly from the stated textual evidence. Eliminate answers
that require assumptions, generalizations, and extrapolations to connect a detail to an inference. Before we draw an
inference from a detail, we should ensure that we understand that detail (and its various ramifications) thoroughly. An
inference will sometimes make logical extensions using new language, so it is vital to understand the detail before
parsing out any potential inferences. Example below:

Given Statement: China has a GDP of about $4.9 trillion USD. Their GDP is higher than South Korea's but not as
high as Japan's.

Possible Inferences:

Valid inference: Japan's GDP is higher than South Korea's.

If China's GDP is higher than South Korea's, and Japan's is higher than China's, it follows logically that Japan's must
be higher than South Korea's.

Ivy-GMAT Exclusive GMAT-Prep for 99th percentile (760-800 range) aspirants not for everyone! Dont join us if you are
not willing to put in the requisite amount of work.
Valid inference: Japan's GDP is over $4.9 trillion USD.

If China's GDP is $4.9 trillion USD and Japan's is higher, it follows logically that Japan's GDP must be higher than $4.9
trillion USD.

NOT a valid inference: Japan exports more products than China.

Whether or not this is true, exports are not mentioned anywhere in the statement, and Japan's higher GDP could
come from any number of other sources of income.

NOT a valid inference: Japan has the highest GDP in Asia.

Even though this is true, it does not logically follow from the information in the given statement. The statement only
compares three countries and does not negate the possibility that another Asian country's GDP might be higher.

Check the example below:

To this day, geologists are divided on the issue of how barrier islands, such as those found off the coasts of
New York, Texas, Mississippi, and Alabama, first formed. The major competing theories can be divided into
three basic groups, each with its origin in the 19th century: the offshore bar theory, the spit accretion theory,
and the submergence theory. Many modern geologists point out that there are examples, even within the
United States alone, that both support and refute each of these three theories. Nevertheless, the scientific
community has not dismissed any of these theories as completely useless. Which of the following can be
most properly inferred from the passage above?

Possible correct answer:

All of these theories explain the formation of at least some barrier islands, but none of them explain them all.

This follows logically from the fact that scientists recognize evidence that both supports and refutes each of these
theories and the fact that scientists have not outright rejected these theories.

Possible incorrect answer:

All geologists believe at least one of these three theories.

The text says that these are the three basic categories into which all major competing theories can be grouped. It is
entirely possible, according to the passage, that a small group of geologists believes in a fourth theory or none at all.

Consider the following passage:

Vincent Van Gogh died penniless and unknown in 1890. Following his death, Van Gogh became gradually more
famous among influential art critics in Europe. His work was exhibited in galleries in New York City, Amsterdam, Paris,
and Cologne, between the years 1900 and 1915. His work influenced a generation of artists, and by the middle of the
20th century, he was known as one of the greatest painters in history. His original paintings are worth millions of
dollars today.

There are a number of inferences we can make from this passage. The passage tells us that Van Gogh died in
obscure poverty but became famous after his death.

We can infer is this: It is possible to die penniless and unknown as an artist and, yet, become famous after death.

Ivy-GMAT Exclusive GMAT-Prep for 99th percentile (760-800 range) aspirants not for everyone! Dont join us if you are
not willing to put in the requisite amount of work.
Lets see the example below:

While effective exploitation of recent technological breakthroughs has been a winning strategy for the
entertainment industry throughout the last century, a number of major entertainment industries are in danger
of near-extinction at the hands of the same technological forces. As evidenced by the evolution from record
album to cassette to compact disc, the music recording industry has beenuntil nowconsistently able to
find a new, even more profitable products to replace those that had been rendered obsolete. However, there
exists no such solution to the complications arising from the recent rise in easily shareable digital music
formats.

Although some experts point out that the increased availability of music will lead to higher profits for record
companies in the long run, many within the industry are skeptical. It may be true that this increased
availability will garner more consumers for artists that are represented by record companies; still, it is unlikely
that such consumers will purchase compact discs while inexpensive digital alternatives are available. Some
companies have had limited success selling digital music one song at a time, but this strategy does not
generate nearly as much profit as have other methods in the past. A decade ago, a music consumer could
have purchased a full-length compact disc for $15, and now he or she is far more likely to purchase only
three or four of its songs for around $1 each, thereby incurring a massive loss in income for the record label.

Which of the following can be properly inferred about music consumers from the second
paragraph of the passage?
A. Technologies that are profitable for record companies do not increase the number of music consumers that
buy the music of a single artist.
B. Music consumers once purchased full-length compact discs when they may have been satisfied with only a
few songs.
C. Technological breakthroughs in the music recording industry in the past have not decreased record company
profits.
D. Compact discs were once at least as popular among music consumers as were cassette tapes.
E. If record companies raised their prices for individual, digital songs, consumers would continue to buy them at
the current rate.

The question stem refers both to a paragraph and a topicmusic consumers. Re-read the sections of the paragraph
that refer to consumers before examining the answer choices.

Choice B is correct. Because the second paragraph mentions that fans who previously purchased full-length CDs for
$15 now often purchase only 3 or 4 songs from those CDs for $1 each, we can logically conclude that, in the past,
consumers must have been buying full-length CDs just to listen to 3 or 4 songs. We can infer that those customers in
the past would likely have been satisfied with the option of purchasing only those songs that they wanted.

Be sure to read question stems carefully: some of these choices (such as choice C) may be inferred from other parts
of the passage but not from the second paragraph.

Ivy-GMAT Exclusive GMAT-Prep for 99th percentile (760-800 range) aspirants not for everyone! Dont join us if you are
not willing to put in the requisite amount of work.
Consider the following passage:

While effective exploitation of recent technological breakthroughs has been a winning strategy for the entertainment
industry throughout the last century, a number of major entertainment industries are in danger of near-extinction at
the hands of the same technological forces. As evidenced by the evolution from record album to cassette to compact
disc, the music recording industry has beenuntil nowconsistently able to find a new, even more profitable
products to replace those that had been rendered obsolete. However, there exists no such solution to the
complications arising from the recent rise in easily shareable digital music formats.

Although some experts point out that the increased availability of music will lead to higher profits for record
companies in the long run, many within the industry are skeptical. It may be true that this increased availability will
garner more consumers for artists that are represented by record companies; still, it is unlikely that such consumers
will purchase compact discs while inexpensive digital alternatives are available. Some companies have had limited
success selling digital music one song at at time, but this strategy does not generate nearly as much profit as have
other methods in the past. A decade ago, a music consumer could have purchased a full-length compact disc for $15,
and now he or she is far more likely to purchase only three or four of its songs for around $1 each, thereby incurring
a massive loss in income for the record label.

Which of the following can be properly inferred about music consumers from the second paragraph of
the passage?
(A) Technologies that are profitable for record companies do not increase the number of music consumers that buy
the music of a single artist.
(B) Music consumers once purchased full-length compact discs when they may have been satisfied with only a few
songs.
(C) Technological breakthroughs in the music recording industry in the past have not decreased record company
profits.
(D) Compact discs were once at least as popular among music consumers as were cassette tapes.
(E) If record companies raised their prices for individual, digital songs, consumers would continue to buy them at the
current rate.

The question stem refers both to a paragraph and a topicmusic consumers. Re-read the sections of the paragraph
that refer to consumers before examining the answer choices.

Choice B is correct. Because the second paragraph mentions that fans who previously purchased full-length CDs for
$15 now often purchase only 3 or 4 songs from those CDs for $1 each, we can logically conclude that, in the past,
consumers must have been buying full-length CDs just to listen to 3 or 4 songs. We can infer that those customers in
the past would likely have been satisfied with the option of purchasing only those songs that they wanted.

Be sure to read question stems carefully: some of these choices (such as choice C) may be inferred from other parts
of the passage but not from the third paragraph.

Ivy-GMAT Exclusive GMAT-Prep for 99th percentile (760-800 range) aspirants not for everyone! Dont join us if you are
not willing to put in the requisite amount of work.
Consider the following passage:

In recent years, historians have struggled to differentiate between historical revisionism, which seeks to reinterpret
orthodox views by considering new evidence and tones, and historical negationism, an illegitimate movement that, in
an attempt to support a particular ideology, seeks to deny the existence of historical facts. On behalf of the American
Historical Association, historians such as Pulitzer Prize-winner James McPherson argued that widely accepted views of
history are fraught with their own biases and must not be considered immutable truths. Rather, McPherson argued,
they should be understood as part of a "continuing dialogue between the present and the past."

Contemporary historians Deborah Libstadt and Alex Grobman are correct to distinguish between this sort of legitimate
revisionism and outright negationism by pointing out the fact that legitimate revisionism acknowledges and respects a
"certain body of irrefutable evidence" with regards to the occurrence of historical events, whereas negationists "reject
the entire foundation of historical evidence." Therefore, a scholar who argues against the racial biases of the Dunning
School's view of post-civil war reconstruction in light of newly unearthed primary source evidence should be
considered a legitimate revisionist, while a scholar who denies that plantation slavery was ever widespread in the
United States should be considered a negationistand, thus, an illegitimate historian.

Based on the passage above, the author would most likely agree with which of the following
statements?
(A) Historical orthodoxies should always be considered true until new evidence arises to call them into question.
(B) Historical revisionists are primarily motivated by ideology rather than academic curiosity.
(C) The Dunning School is an example of a group of scholars engaging in legitimate historical revisionism.
(D) While he or she can question orthodoxies, a revisionist historian must accept certain facts in order to be
considered legitimate.
(E) Contemporary historians have become increasingly tolerant of negationist theories in light of recent historical
discoveries.

The question stem does not direct us to a particular part of the passage, so we should move directly to the answer
choices, eliminating those with which the author would not likely agree.

Choice A: The word "always" signals that this is an extreme answer choice, and in fact, the author would certainly not
necessarily agree that historical orthodoxies should be taken at face value. The phrase "new evidence" refers to the
end of the passage, so we can take this cue to revisit that section of the text.

Choice B introduces an irrelevant comparison; "academic curiosity" is never discussed in the passage.

Choice C refers to a specific claim in the passage, which we should revisit: a scholar who argues against the racial
biases of the Dunning School's view of post-civil war reconstruction in light of newly unearthed primary source
evidence should be considered a legitimate revisionist. Choice C mistakes one group for another; the author suggests
that the scholar who studies the Dunning School in this hypothetical example is a "legitimate historical revisionist,"
not that the scholars in the Dunning School are.

Choice D refers to "legitimacy," and in fact, the author states in the first paragraph that negationismis an illegitimate
movement which seeks to deny the existence of historical facts and again, at the end of the passage, that those who
deny facts are illegitimate revisionists. Choice D is correct.

Choice E is an opposite choicethis author is staunchly against negationism.

Ivy-GMAT Exclusive GMAT-Prep for 99th percentile (760-800 range) aspirants not for everyone! Dont join us if you are
not willing to put in the requisite amount of work.
Consider the following passage:

Fitness center managers who hope to maximize profits have long known that one of the best ways to attract new
members is to use personal trainers as salespeople. Between workout sessions, trainers spend a significant amount of
time giving free advice to prospective clients and offering tours of the facilities. Managers and trainers are happy with
this arrangement, as it attracts new customers to the fitness center and provides low-stress, fully-paid rest time for
the trainers.
Which of the following is best supported by the information above?
(A) The hourly cost of paying personal trainers to act as sales staff is less than the revenue generated by the increase
in membership fees paid by the newer gym users.
(B) Because the work of personal training offers such low pay for high-stress physical labor, trainers are only willing to
exercise with clients for a specified number of hours per day.
(C) Fitness center managers have to put the interests of prospective clients above those of trainers, who would rather
not work as salespeople.
(D) Most prospective gym members prefer not to talk to dedicated salespeople and would rather get free advice from
a professional trainer than spend money on a gym membership.
(E) Each new membership that a gym earns generates enough revenue to cover the cost of paying a single trainer to
give free tours and advice for that day.

This question asks us to derive a general principle from a specific case. In this instance, we are not looking for a very
far-ranging abstraction; the answer will refer to some basic idea about gym memberships and free trainer tours.
Choice A is correct. The other choices are too extreme (B, the word "only" kills it), potentially true on their own but
not implied by the passage (E, D), or inaccurate inferences (C, the passage states that trainers are happy with the
arrangement).

Ivy-GMAT Exclusive GMAT-Prep for 99th percentile (760-800 range) aspirants not for everyone! Dont join us if you are
not willing to put in the requisite amount of work.
Consider the following passage:

In the business world, many tend to consider technological innovation the main method by which a company comes
to dominate a given market. Though this may seem intuitive, and though many examples can be unearthed in
support of this maxim, it is not entirely true. In fact, it has often been the case that a company that has come to
dominate a given market has reached that position by improving upon and rendering obsolete some existing
technology. To the extent that patent law allows, a company can build upon the inventions of othersa practice
resulting in a dramatic reduction of that company's development costs. In this same vein, a company can make
improvements to a product that are sufficiently significant to shift the market decisively in that company's own favor.

The Alphamax video cassette recorder, a device developed by Company X in the mid 1970s, is a classic example of
this phenomenon. At the time of its inception, the new recorder could boast a VHS format that was considered a
revolutionary improvement over the previously ubiquitous 8mm recording devices that had been used for home
movies and other amateur recordings. This new recorder enjoyed only two short years of widespread use, however,
before Company Y introduced the VBD recordera recorder which quickly proved more profitable. Company Y's desire
to introduce a product that was extremely similar to Company X's new recorderbut different enough to avoid
violating any patent lawsinduced Company Y to create a recorder that offered longer playback and more convenient
recording than the one put forth by Company X. Company Y's recorder featured tapes with large capacities that, given
the advent of full-length rental films, no longer seemed superfluous. Because Company Y was able to create a product
that was both similar to and that improved upon Company X's existing product, Company X's existing product soon
became nearly obsolete.

It can be inferred from the passage that consumers came to prefer VBD tapes for which of the following
reasons?
(A) They were sufficiently different from Alphamax tapes to be offered to consumers without any possible risk of
patent law violation.
(B) They were introduced at a time when consumers were very fickle and eager to consume new products.
(C) More movies were available for rent in VBD format than in the Alphamax format.
(D) VBD tapes utilized a video home-recording and playback technology that offered distinct advantages when
compared with Alphamax.
(E) Companies with experience in the field of video recording began to move from the Alphamax market to VBD,
bringing with them technological and marketing expertise.

Choice D is correct. The evidence that VHS overtook the market previously dominated by Alphamax, combined with
the statement that VHS offered longer playback and more convenient recording, is sufficient to allow us to infer that
consumers preferred VBD technology because it was a new product that was both similar to one they had already
embraced and distinct enough to offer new advantages. Note that although choice A is true according to the passage,
it does not explain why consumers preferred VBD tapes to VHS (it is likely that consumers are not terribly concerned
about patent law violations).

Ivy-GMAT Exclusive GMAT-Prep for 99th percentile (760-800 range) aspirants not for everyone! Dont join us if you are
not willing to put in the requisite amount of work.
Consider the following passage:

The practice of deploying civilian journalists into combat zones within a troop of soldiers is known as "embedding."
Though the practice dates back to the 19th century, official military and journalistic standards have only been applied
to the practice since the Cold War. The Vietnam War was the conflict in which a large number of television, radio, and
print news reporters traveled into the combat zone alongside soldiers. Today, most cable news stations rely heavily on
embedded journalists for on-the-ground reporting and seek to expand the practice into more conflict areas around the
world. Whether or not this comes to pass will depend upon how military officials view the practice.

Non-commissioned officers and lower ranked soldiers tend to see the practice of embedding in a favorable light.
Informal surveys suggest that these soldiers respect reporters willing to put themselves in harm's way and feel that
such reporters are in the best position to accurately represent a conflict and tell their stories. Higher ranking officers,
such as colonels and lieutenants, tend to be more suspicious of such journalists. They are not as quick to grant
interviews or supply information to embedded journalists, ostensibly out of fear of divulging information that may be
used by enemies. Many media professionals feel that these upper level officers have more to lose from negative press
and are more concerned with their future political ambitions than with accurate war reporting.

It can be inferred from the passage that the media professionals mentioned in the second paragraph
would most likely agree with which of the following statements?
A. Military officials need to be convinced of the value of embedded journalists so that the practice can be expanded
into more conflict zones around the world.
B. It is likely that army generals sometimes use the fear of disclosing potentially sensitive information to the enemy
as a pretense for censoring embarrassing information.
C. Since there were no officially embedded journalists prior to the Vietnam War, the news reports regarding prior
wars are likely inaccurate and heavily biased.
D. Because so many of them intend to run for political office upon completing their service, high ranking officers are
easy to interview, as they appreciate the opportunity to tell their own stories.
E. Enlistees and low rank officers tend to be less reliable sources for news articles, as they tend to be overly
concerned with future political ambitions.

Choice B is correct.
We know that the "ostensible" reason for blocking embedded journalists from access to information is fear of
unintentionally aiding the enemy. We also know that Many media professionals feel that high level officers only care
about blocking negative press for the sake of their political ambitions. With the qualification would most likely agree
with softening the blow, choice B connects these two ideas and is the most logical answer.
The other answer choices are either opposite (D), potentially true but not implied (A & C), or pertain to the wrong
group (E).

Ivy-GMAT Exclusive GMAT-Prep for 99th percentile (760-800 range) aspirants not for everyone! Dont join us if you are
not willing to put in the requisite amount of work.
Consider the following passage:

Young children are advised to consume Vitamin D, a necessary ingredient for the regulation of calcium and phosphate
in the bloodstream, in order to strengthen their immune systems and avoid bone disorders like osteomalacia. Vitamin
D is found both in fatty fish (including salmon, tuna, and mackerel) and in beef liver.
Which of the following statements can be inferred from the passage above?
(A) Children who do not consume large amounts of fatty fish rich in Vitamin D run a high risk of developing bone
disorders.
(B) Seniors should consume fatty fish and beef liver to strengthen their immune systems.
(C) Vitamin D is naturally produced by the skin when the body is exposed to sunlight.
(D) Young children can reduce the risk of acquiring osteomalacia by consuming tuna or salmon.
(E) It is safer to obtain Vitamin D from natural sources such as fish and beef than it is to take Vitamin D supplements.

We need to be careful of answer choices that fall into the "could be true but not implied by passage" category.
Inference questions frequently devise plausible-sounding answer choices as distractors. Remember, a statement that
has no roots in the text cannot be considered an appropriate inference. Calling upon outside knowledge often leads
test-takers astray.

Such is the case with answer choice C. Though choice C states a true fact, that detail is not suggested by any of the
information in the passage. Choices E and B both might be true, but nothing in the passage refers to seniors or
vitamin D supplements; therefore, even though both are probable, neither can be inferred from the text.

This leaves us with answer choices A and D. If we think about them carefully, it should be clear that D is the more
logical inference. Osteomalacia is a bone disorder. Children can avoid bone disorders by consuming Vitamin D.
Vitamin D is found in fatty fish. Tuna and salmon are both fatty fish. By syllogism we can deduce: Young children
consuming tuna and salmon reduced risk ofosteomalacia. Choice D is correct.

Choice A is a tricky distractor, because it seems to be vaguely implied by the excerpt; ultimately, however, choice A is
too extreme. The phrases "large amount" and "high risk" make this too strong to be a natural consequence. Also, just
because Vitamin D helps prevent bone disorders, it does not follow that not consuming Vitamin D generates a "high
risk" of getting bone disorders; other foods may be equally helpful in the reduction of bone disease, and it is plausible
that a person might stop taking Vitamin D in order to increase intake of some other bone-beneficial substance.

Ivy-GMAT Exclusive GMAT-Prep for 99th percentile (760-800 range) aspirants not for everyone! Dont join us if you are
not willing to put in the requisite amount of work.
Identifying author's tone

Consider the following passage:

The ability of the FDA to adequately monitor the information transmitted by drug companies to consumers through
advertisements is, at best, questionable. According to a 2006 report by the United States Government Accountability
Office, the FDA reviewed only a small portion of the advertising materials it received, and the organization could not
confirm that the materials that underwent this review were those that had been identified as higher priority
advertisements. Though the study's specific claims might be disputed, the broader implications of such a finding
should be clear: advertising materials from drug companies cannot be trusted.

In this passage, the author begins by asserting the view that the FDA's ability to monitor drug company ads is
questionable.

The second sentence begins with according to..., which indicates that the statement that follows reflects another
party's toneone which is distinct from the author's (in this case, the other party is the United States Government
Accountability Office). The final sentence, which points to the possible unreliability of the government office's study's
claims, returns to the author's point of view.

Certain details which can be inferred from the passage may be more reflective of one tone than another; it is
important to attribute these inferences to the correct party. Consider the following inference:

The FDA accounts for the fact that it monitors only a small portion of drug company ads by attempting to identify high
priority advertisements, though it is not always successful in this endeavor.

This loose inference aligns with the tone of the United States Government Accountability Office; the second sentence
suggests that at least some material had been identified by the FDA as higher priority advertisements, though the
FDA could not confirm that it was these high priority materials that had been reviewed. The author, however, might
take issue with the information contained in this inference, as she mentions that the study's specific claims might
be disputed.
People should not view drug company ads as reliable sources of information about a drug.

Since it is specifically the author who claims that the FDA's monitoring abilities are questionable and that advertising
materials from drug companies cannot be trusted, this inference describes a sentiment that the author likely believes.
The author cites a study conducted by the the United States Government Accountability Office in order to lend
credence to this point, but we cannot assume that the United States Government Accountability Office would have
drawn the same conclusion from its own study. In other words, this inference does not necessarily reflect something
that the Accountability Office might think.

Consider the following passage:

Urban planning in this county has increased in complexity of late, as an increasing number of environmental and
political concerns have arisen. Though many citizens indicate that urban development should accelerate to meet the
demands of a growing population, this short-term mentality is ultimately destructive. Urban development should be
submitted to a slower, more bureaucratic process.

In this example, the author's tone and the tone of "many citizens" are in conflict.

Ivy-GMAT Exclusive GMAT-Prep for 99th percentile (760-800 range) aspirants not for everyone! Dont join us if you are
not willing to put in the requisite amount of work.
Consider the following passage:

The new regulations proposed for the interstate trucking industry are convoluted and nonsensical, as they are
burdened with rules that are difficult to follow and do little to increase safety. These rules constitute a prime example
of frivolous government regulation gone completely overboard. All sensible citizens should write to their
representatives and state their grievances.

The author's main idea, here, is that the interstate trucking regulations are a bad idea; likely, the
author's purpose in this piece is to express this opinion. A tone question about this excerpt will read slightly
differently than a main idea or purpose question. The following, for example, asks about the author's tone:
Government regulations in general seem to be regarded by the author with: A possible answer to such a question
might read: Overt disdain.

Consider the following passage:

Montgomery's analysis of seventeenth century trade practices provides the best available analysis of intercontinental
trade policy throughout Asia and the Middle East; her thesis about the creation of preservatives to transport
perishable goods is particularly compelling. However, she refuses to contend with the inherent difficulties in studying
trade histories written by partial, nationalist observers.

In this sample, there are two tones: that of "Montgomery" and that of the author. They agree on some aspects of the
argument (the creation of preservatives) and disagree on others (partial observers may be less trustworthy). Some
GMAT passages compare the tones of two different passage sources; in these cases, three tones must be noted.

GMAT passages tend to take on a characteristically neutral, journalistic tone.

Example:

Current scholarship on Romantic literature neglects the vital contributions of female writers. For this reason, important
texts that contain first-hand knowledge of early 19th century life written by females of the period are oxidizing in
moldy libraries and are slowly disappearing. This is a tragedy which should be addressed immediately by the
establishment of annual conferences and professional organizations that focus on the achievements of women writers
of the Romantic period. With which of the following statements about women writers of the Romantic
period would the author most likely agree?
A. They are growing in importance and should be addressed by the establishment of more professional
organizations.
B. More professional organizations devoted to protecting their work from oxidization are necessary.
C. Their achievements have not been fully appreciated by modern scholars.
D. Current scholarship on Romantic literature focuses on only their most vital contributions to literature.
E. Annual conferences and professional organizations should be established to advocate greater interest in writing
among women.

The author's tone on Romantic-period women writers is that current scholarship...neglects [their] vital contributions.
All four wrong answer choices for this question attempt to use terms from the passage to confuse the test-
taker. Choice C is correct.

Ivy-GMAT Exclusive GMAT-Prep for 99th percentile (760-800 range) aspirants not for everyone! Dont join us if you are
not willing to put in the requisite amount of work.
Consider the following passage:

Despite the emergence of a trend favoring flatter organization of human capital, companies should seriously consider
the possibility that more rigid hierarchies might be better-suited to their purposes. The current management strategy
at Globatron, for instance, involves a dependency on the superior talents of its younger employees. These newer
employees, Globatron noticed, often have the capacity to solve complex management issues that would ordinarily fall
under the aegis of upper management. It remains to be seen whether the company will continue to perform well
when its relatively inexperienced leaders encounter issues that require significant depth and breadth of industry
knowledge. What is the author's tone on the current trend toward flatter organization of human capital?
A. Other companies should emulate the hierarchy of human capital that has been instituted at Globatron.
B. Upper management should always resolve issues that require significant depth and breadth of industry
knowledge.
C. Globatron's current strategy is vulnerable to failure on the grounds that young leaders are likely to encounter
problems that require more experience than they can claim.
D. A company should take careful consideration when deciding which organization of human capital is most
appropriate for its purposes.
E. Flat organizations of human capital are generally superior to more rigidly hierarchical ones.

Choice D is correct. The author's tone on the cited general trend is mentioned in the first line: companies should
seriously consider the possibility that more rigid hierarchies might be better-suited to their purposes. This is the
statement of opinion put forth by the author; the example involving Globatron and the reference to weaknesses in its
strategy both support the author's general stance that rigid hierarchies should not be dismissed out of hand
despite the general trend (conventional wisdom) favoring the opposite. Note that choices A and C are too specific, as
they refer to tones on Globatron in particular and not to the trend that can be observed in companies as a whole.

Consider the following passage:

The recommendations of our political counsel have been faulty. We should not implement their ideas despite
unanimous approval of their proposal by the board of trustees, which believes the proposal will be effective and,
moreover, that it will unite the team by reflecting a variety of interests. I insist that the proposal is rudimentary, fails
to address the most important issues at stake, and opts instead to present an optimistic picture that is dangerous in
its naivet. What is the board of trustees' tone concerning the recommendations of the political counsel?
(A) It is more important that a proposal unite the entire team than that it be highly effective.
(B) The proposal is rudimentary and naive.
(C) The recommendations of the political counsel will be effective and unite the whole team.
(D) The board of trustees will be more effective than the political counsel.
(E) The ideas of the political counsel are flawed and should not be implemented.

Choice C is correct. It clearly states in the passage that the board of trustees believes the proposal will be effective
and will unite the team by reflecting a variety of interests. Here, it is crucial to distinguish the tone of the author from
that of the board of the trustees (since the question concerns the trustees' opinion, not the author's).

Words such as "naive" and "dangerous" reflect a staunchly negative tone, and, thus, are incorrect. If we recall
positive passage words such as effective and unite and associate them with the source "board of trustees," we will be
able to eliminate wrong answer choices rather immediately.

Ivy-GMAT Exclusive GMAT-Prep for 99th percentile (760-800 range) aspirants not for everyone! Dont join us if you are
not willing to put in the requisite amount of work.
Consider this example:

Recently, an increasing number of K-12 schools, both public and private, have instituted a policy stating that all
students must wear uniforms to school. Administrators' reasons for this decision are two-fold: they believe the
uniforms will aid in the reduction of violent bullying on school property and will diminish the effect of economic class
as a divisive factor in social situations. Some parents' views align with the school boards' on this matter. Not only are
these parents thankful to be relieved of the burden of back-to-school clothes shopping, but they are also hopeful that
a mandatory formal attire will instill a sense of discipline and school pride in the students. The much-touted academic
successes of Japanese students, all of whom are required to wear uniforms, seems to bolster optimism regarding this
practice.

Alas, reality has a way of rudely disabusing us of such lofty hopes. In truth, uniforms do not account for Asian
students' achievementstheir communal meritocratic culture, more likely, is the root of their diligence. Our dog-eat-
dog competitive individualist society, in which one student's success is seen as everyone else's failure, differs
astronomically from the Japanese model. In point of fact, uniforms have been standard in inner-city "last chance"
schools for decades, and no measurable improvement has been the result. It is no surprise, then, that surveys of
students at schools that require uniforms consistently show more pronounced clique-forming behaviors and more
persistent discipline problems. The question is: when will the adults in charge start to listen?

Three distinct tones are associated with this passage; the tone of the first paragraph, the tone of the second
paragraph, and the tone of the overall passage.

First paragraph: The tone of this paragraph is calculated and objective. The author does not take a stance on
the controversial issue that has been presented; rather, she presents the various viewpoints in support of the trend,
careful to attribute each opinion to a specific source ("administrators" and "parents"). The word "seems," however, is
a chief (and subtle) clue that the author's contrasting view is on the horizon.

Second paragraph: The second paragraph immediately converts to a polemical, almost contemptuous voice, using
melodramatic words like "alas"not to mention the sarcastic phrase "lofty hopes." Notice the greater freedom with
figures of speech, like "last chance" and "dog-eat-dog." Notice also the assertive language of debate, phrases like "in
point of fact" and "it is no surprise." The rhetorical question at the end accentuates this paragraph's overall tone
of passionate disdain, or even outrage.

Both: Together, the tones of these paragraphs make sense. The passage introduces a topic and describes the
opposing viewpoint; then, the author dismantles this opposing tone with both logic, hard evidence, and a bit of
sardonic rhetorical flourish. The overall passage tone is therefore best characterized as confident certitude.

Tone of Single Words and Phrases:

1. Baseball lacks the dynamism and physicality of football, which makes baseball an inferior sport.

2. Football is the king of sports because it is fast-paced, tactical, and violent, while baseball is about as exciting as
watching grass grow.

3. One has no choice but to admit that, when it comes to sheer athleticism and action, football has something of an
edge over the more measured, slowly simmering pace of baseball.

Notice that each sentence makes the same point: Football is more exciting than baseball. It is thetone of each
sentence that differs.

The first sentence displays a dispassionate certainty, with its sophisticated vocabulary ("dynamism" and
"physicality") and direct conclusion.
The second sentence takes on an irreverent attitude, as evidenced by its use of a humorous comparison and
figurative language.
The third sentence shows a begrudging acceptance, evident in its qualifying language ("something of an edge")
and euphemistic characterization of baseball.

Ivy-GMAT Exclusive GMAT-Prep for 99th percentile (760-800 range) aspirants not for everyone! Dont join us if you are
not willing to put in the requisite amount of work.
Consider the following passage:

By way of new promotional efforts, increased attention to its social media program, and the inauguration of a newly
elected board of directors, the company is attempting to achieve its laudable goal of capturing an even greater
portion of the market; still, a more direct focus on the customer would perhaps be a more likelyand far less
expensivemeans of achieving similar effects.
The author's use of the word "perhaps" in this statement might be characterized as conveying a tone of
(A) indignation
(B) admiration
(C) confusion
(D) indifference
(E) skepticism

Choice E is correct. Positive words and phrases in the first section of the passage, such as cutting-
edge, laudable, and undoubtedly, are offset by the claim in the last section of the passage, in which the company's
methods are questioned. Since the tone is fairly neutral, choices like "indignation" and "admiration" are too strong.
"Skepticism" effectively conveys the author's mixed attitude of consideration and doubt.

Don't expect especially passionate or emotional language on the GMAT; on the contrary, sometimes the tone of a
GMAT passage is very subtle. Rather than looking for extreme, overtly emotional language, we should take our tone
signals from words like the following:
Fortunately
Unfortunately
Preferable
Advantage
Defend
Foster
Victim
Triumph
Unrealistic
Protest
Appreciate
Instigate
Optimistic
Satisfy
Shortcoming

Ivy-GMAT Exclusive GMAT-Prep for 99th percentile (760-800 range) aspirants not for everyone! Dont join us if you are
not willing to put in the requisite amount of work.
Consider the following passage:

Manager: The extent to which this company lacks transparency is disturbing. Senior management determines our
overarching strategy, and those at the lower levels execute itoften with little or no sense of the larger context. This
practice, which purposely prevents certain employees from developing their own opinions about company strategy,
severely limits our growth as a company.

The efforts of our middle management, on the other hand, are phenomenal. Their establishment of both a mentoring
program and a two-month training program for new employees reflects a serious investment in human capital. The
culture of our workplace is significantly improved by their vision.

The manager's tone in the passage might be characterized as


(A) Bitter and resentful
(B) Proud and admiring
(C) Critical and balanced
(D) Engaged and sympathetic
(E) Moderate and cautious

Choice C is correct. In the first paragraph, words such as disturbing, severely, and limits indicate that the tone is
extremely critical. In the second paragraph, however, words and phrases such aspraise, serious investment,
and significantly improved suggest an admiring tone. Because the manager exhibits differing tones towards two
different levels of management, her tone can be said to be "balanced."

Identifying purpose

Several answer choices can be quickly eliminated on such questionsmany purpose answer choices contain at least
one of the following errors:

Too Narrow In Scope: Some answer choices refer to one section or paragraph in the passage, but not to the
passage as a whole.
Too Broad in Scope: Some answer choices incorrectly refer to a broad, vague topic. If a passage is about a
specific historical time period, for example, an incorrect answer might refer to "history" or "the past."
Extreme: Some answer choices overreach, claiming that an author intends to "prove" or "disprove" a topic
that is merely being introduced and gently explored. Most GMAT passages do not express strong opinions.

Common Neutral GMAT purpose verbs include:

Explain no opinion
discuss no opinion
describe no opinion
illustrate no opinion
compare no opinion
examine no opinion
imply no opinion
suggest opinion
consider no opinion

Common Positive GMAT purpose verbs include:

defend opinion
validate opinion
support opinion
advocate opinion
recommend opinion
argue for opinion

Common Negative GMAT purpose verbs include:

undermine opinion
Ivy-GMAT Exclusive GMAT-Prep for 99th percentile (760-800 range) aspirants not for everyone! Dont join us if you are
not willing to put in the requisite amount of work.
warn opinion
dispute opinion
dispel opinion
critique opinion
criticize opinion
deny opinion
argue against opinion

Consider the following passage:

Among manufacturers who sell similar products, it has become a progressively more common practice to undercut
competitors' prices by offering discountsin an effort, of course, to attract consumers. Many economic studies show
that this practice actually ends up reducing revenue for the companies involved. Unfortunately, many executives still
ignore these studies and continue discount practices that damage their profit potential.

What is the primary purpose of this passage? Possible correct answer:

To warn against the use of discounts to increase earnings.

Words like unfortunately make it clear that the author has a decisively negative view of discount practices, so the verb
used to describe the primary purpose should also be negative.

Consider the following passage:

Many science historians have independently noted that the concept of "heat" is commonly misused outside of the
science world. While many people may know that heat is a measure of energy, most fail to realize that an object or
substance cannot technically contain its own measure of heat, because heat is a measure of the energy transferred
from one object to another. A piece of metal that feels cool to an observer (who has been sitting in a room of equal
temperature) does not lack heat; the "cool" feeling is simply a result of the fact that the object is pulling heat away
from a significantly warmer bodythe observer himself. This is a particularly egregious mistake, since understanding
"heat" is critical to maintain a grasp of the laws of thermodynamics that regulate an enormous percentage of the
machines and systems we encounter on a daily basis. Thermodynamicsthe study of energy conversion from heat to
mechanical workis such a valuable field, partially because its underlying premises have been so badly misconstrued.

Possible answers:

To explain a commonly misunderstood concept


To relate a misunderstanding to the value of a field of study
To discuss the confusion surrounding a scientific idea
To explain the concept of heat and its implications for the understanding of the laws of thermodynamics

Consider the following passage:

Industries are facing increasing global competition, in addition to political and consumer pressure, to move toward
streamlined, sustainable, and innovative technologies. Traditional reliance on processes requiring high temperatures,
pressures, or harsh chemical additives as reaction catalysts has resulted in high energy consumption and in the
release of harmful byproducts that require significant financial investments to address. As environmental concerns
gain in popularity, industrialization in emerging regions continues, and energy costs continue to soar. Biological tools
are increasingly replacing harsh chemical and physical means of synthesizing chemicals and processing materials, and
industries are striving to uncover cost-effective, renewable energy and chemical product alternatives to those derived
from petrochemical sources.

Ivy-GMAT Exclusive GMAT-Prep for 99th percentile (760-800 range) aspirants not for everyone! Dont join us if you are
not willing to put in the requisite amount of work.
Possible answer:

To describe several reasons for the increased adoption of biological synthesis tools
To explain the increasing preference for one scientific innovation over others

Purpose / Role of specific paragraphs:

Consider the following passage:

The need to derive more of our energy from renewable sources is now almost universally acknowledged. Due to the
growing costs of traditional energy sources like oil and coal, the environmental impact of CO2 emissions, and the
geopolitical implications of dependence on foreign energy, the benefits of moving towards energy that is clean,
renewable, and domestically produced are obvious. The main obstacle impeding adoption of more renewable energy
sources involves the short-term costs of installation of these new energy sources which, despite widespread
knowledge of the possible long-term savings, can be high enough to be considered prohibitive. The question, then, is
whatbeyond the simple promise of long-term savingswill encourage energy companies and consumers to weather
these short-term high costs.

Many disagree, in particular, about the potential implementation of investment subsidiessubsidies paid by the
government to energy producers to defray the cost of installation of solar panels, wind turbines, and other sources of
renewable energy. Proponents say that such subsidies are a unique way to encourage private companies, which are
generally focused on their short-term revenue, to make economic decisions that benefit both themselves and the
public in the long run. Critics counter that it is unfair to distribute the burden of a subsidy equally among taxpayers,
without regard to each taxpayer's individual energy consumption.

A more equitable policy may be the enactment of feed-in tariffs. Feed-in tariffs are a unique type of agreement by
which electric grid and other public utility companies agree to buy excess energy produced by a clean and/or
renewable source on a multi-year contract at a guaranteed rate. This enables potential investors in renewable energy
to see a faster and more reliable profit than they would otherwise, while proportionally distributing the financial
burden among the utility consumers according to consumption. Many such programs have been successful in recent
years, particularly in areas with high levels of sunshine and high electricity costs, including California and Japan.

The main purpose of the third paragraph of this passage is to


(A) introduce a controversy
(B) argue against an opponent's proposal
(C) propose and evaluate a solution
(D) describe a problem and offer a solution
(E) reconcile two alternative proposals

We should approach this question with an eye towards the purpose of the passage as a whole; specifically, however,
we will focus on what the author is attempting to accomplish in the third paragraph.

This passage begins by describing a problem and proceeds to evaluate potential solutions to that problem. The
author's opinion is not breached until the beginning of the third paragraph; here, the phrase a more equitable
policy indicates that the author agrees with the critics mentioned in the second paragraph. Evidently, the author
agrees that the energy cost to each individual should be proportional to that individual's energy usage. The third
paragraph expands upon this idea, introducing and describing a solution that conforms to such a model. Choice C
is correct.

Let's take a look at the answer choices, in order: The controversy is introduced in the first paragraph, not
the third, so choice A is incorrect; the third paragraph is spent describing the merits of a potential solution, not
denouncing an opponent's solution, so we cannot pick choice B; choice D is incorrect because it moves beyond the
third paragraph, instead describing the purpose of the passage as a whole; choice E makes a false statement, as the
third paragraph fails to reconcile the two opposing views mentioned in the previous paragraph.

Consider the following passage:

Ivy-GMAT Exclusive GMAT-Prep for 99th percentile (760-800 range) aspirants not for everyone! Dont join us if you are
not willing to put in the requisite amount of work.
Eutrophication, the process by which a body of water acquires a high concentration of nutrients, has become known
to the environmentally aware community as a key human stressor on the world's ecosystems. High concentrations of
nutrients, specifically phosphates and nitrates, promote excess growth of primary producers. Excessive growth of
algae and other simple photosynthetic organisms can deplete water of available oxygen, causing the death of other
organisms inhabiting the water. Eutrophication can sometimes be a natural, slowly developing process, but human
activity, specifically the overuse of fertilizers causing runoff from farm land that distributes nutrients to nearby waters,
is greatly speeding up the problem. This water runoff causes in the local rivers, lakes, and coastal ocean
waters unnaturally high rates of plant production and, eventually, an accumulation of organic matter that degrades
water and habitat quality.

Riparian buffer zones, the interfaces between land and rivers or streams, are significant in many types of
environmental management because of their role in soil conservation, their biodiversity, and their potential influence
on aquatic ecosystems. Riparian zones occur in many forms, including grasslands, woodlands and wetlands; these
surround freshwater, as well as marine ecosystems. Restoring these buffer zones between farm land and damaged
aquatic systems is one step towards the reduction of eutrophic water. A comprehensive solution would also involve
reducing livestock densities, improving the efficiency of fertilizer applications, treating urban runoff from streets and
storm drains, and reducing nitrogen emissions from vehicles and power plants.

Which of the following best describes the role of the second paragraph above?
(A) to highlight the relative advantages of alternative methods to combat a problem
(B) to introduce a specific example of a previously introduced problem
(C) to provide evidence that refutes the main idea of the passage
(D) to explain a theory that is introduced in the first paragraph
(E) to describe methods for solving a problem introduced in the first paragraph

Choice E is correct. The second paragraph describes a number of methods that could provide a solution to the
Eutrophication problem outlined in the first paragraph.

Choice A is probably the most tempting wrong answer choice, as it suggests that the second paragraph highlights the
relative advantages of the methods the author introduces. While several different solutions are presented, however,
they are not compared to each other or to any other methods.

Consider the following passage:

The Consumer Product Safety Commission was created in 1972 to protect the American public against unreasonable
risks of injuries associated with consumer products. The main functions of the CPSC include issuing recalls for
products that have been reported to be unsafe, providing consumer education that assists consumers in evaluating
the comparative safety of consumer products, developing uniform safety standards for consumer products, and
promoting research and investigation into the causes and prevention of product-related deaths, illnesses and injuries.

Product defects must be reported to the CPSC by a manufacturer, distributor or retailer who obtains information that
reasonably supports the conclusion that the product fails to comply with a consumer product safety rule. The
manufacturers, distributors and retailers are responsible for identifying any product that contains a defect that creates
a substantial product hazard or creates an unreasonable risk of serious injury or death. The Commission may
impose civil penalties if there is a failure to report a substantial product hazard to the commission in a timely manner
or if a company willfully fails to comply with a safety standard.

The agency stays informed of unsafe products, partly by way of a consumer hotline and a website through which
consumers may report concerns about unsafe products or injuries associated with products. Unfortunately, the CPSC
often relies on manufacturers of defective products to handle safety issues, and incidents have occurred wherein
several children suffered injury before the CPSC acted on reports about the potential dangers of a product. Many
believe, additionally, that the CPSC is underfunded and understaffed. These critics suggest that the CPSC should be
given more authority to order recalls and fine companies that put dangerous products on the market; these critics
also maintain that the CPSC should be provided with improved product testing facilities.
Which of the following best describes the role of the second paragraph above?
(A) to provide evidence to emphasize the importance of the CPSC
(B) to introduce specific examples of reports that have been made to the CPSC about dangerous products
(C) to describe the process of identification and reporting of hazardous products
(D) to inform the reader of the flaws in the enforcement policies of the CPSC
Ivy-GMAT Exclusive GMAT-Prep for 99th percentile (760-800 range) aspirants not for everyone! Dont join us if you are
not willing to put in the requisite amount of work.
(E) to describe the primary function of the CPSC

Choice C is correct. A quick outline of the passage can be used to easily eliminate a number of wrong choices.
Choice C is the only one that conforms to the logic of the outline.

Consider the following passage:

In the 1980s, new, alternative theories of leadership were developed, many of which heavily focused on statistical
meta-analyses of previous studies and on the gathering of data on successful corporate cultures. This trend resulted
in the emergence of trait theories, which claimed that effective leaders tend to exhibit particular personality traits.
Intelligence, extraversion and conscientiousness, among others, were named by such theories as the defining
"qualities of a leader".
Trait theories, although a major step forward in empirical leadership studies, overemphasized the universality of these
leadership traits. In many leadership roles or situations, humanitarian qualities such as humor, empathy, or generosity
play a far larger role than do the classically defined qualities of leadership observed in statistical analyses. However,
these qualities, which are more personal and difficult to define or report, are far less likely to be analyzed with the
same vigor.
Which of the following best describes the role of the second paragraph in relation to the passage as a
whole?
(A) to introduce an alternative to the theory presented in the first paragraph
(B) to provide a concrete example to support a more general argument made in the first paragraph
(C) to point out a flaw in a theory presented in the first paragraph
(D) to reconcile two conflicting theories discussed earlier in the passage
(E) to apply a previously discussed theory to a different situation

Choice C is correct. The second paragraph, after acknowledging that trait theories (summarized in the first
paragraph) represented a major step forward in empirical leadership studies, points out that these theories
overemphasized certain traits at the expense of other, equally important ones. This constitutes a flaw in the theory
presented in the first paragraph.

Providing definition for word or phrase in context

The reclusive man rarely emerged from his apartment or greeted anyone in the hallway.

The word "shy," for example, would make sense in this context and is fairly close to the definition ofreclusive.

Contrast clues such as "but," "however," and "unlike" tell us that the word we are attempting to define is the opposite
of what the rest of the sentence describes.

Unlike his introverted and unsociable brother, Carl is gregarious

In this example, the context clues tell us that gregarious means the opposite of introverted and unsociable.

A clause in the sentence, set off with commas, may also provide the definition of an unfamiliar word.

Josh hated that his sister, always hyperbolic, exaggerated every story she told.

In this case, exaggerated every story accurately expresses the meaning of the word hyperbolic.

Ivy-GMAT Exclusive GMAT-Prep for 99th percentile (760-800 range) aspirants not for everyone! Dont join us if you are
not willing to put in the requisite amount of work.
A sentence may describe a cause and its effect. If an unfamiliar word represents the cause but the effect is stated in
familiar words, we can infer the meaning of the unfamiliar word.

The strong feelings of antipathy that Julie felt from the other members of her class caused her to skip school

What sort of feelings would cause Julie to skip school? The effect allows us to conclude that the other students'
feelings towards her were negative; therefore, the word antipathy refers to something negative.

Against his better judgment, a plant manager agreed to train his neighbor's delinquent son as an apprentice for a new
position. As the training began, the boy asked numerous questions about every new task he was shown, including the
most simple ones. On his lunch break, the manager said to his assistant, "That new boy is either extremely
unintelligent or deliberately obtuse."

In the context of the passage, the word "obtuse" most closely corresponds to which of the following?

The context clue either extremely unintelligent or... lets us know that being deliberately obtuse is a similar explanation
for this boy needing very simple tasks explained to him. Therefore, we can conclude that being obtuse has something
to do with being dim-witted.

As with this example, there is often a possible definition for the word in question that is more familiar or common but
that would not appropriately fit the context of the passage. In this case, "obtuse" clearly does not refer to an angle
between 90 and 180 degrees!

Occasionally, the GMAT requires the test-taker to provide a synonym for a particular word as it is used in the context
of the passage. These questions will often attempt to confuse the test-taker by providing several answer choices that
are closer to the standard definition of the word but do not take into account the specific context in which it appears.

Consider the following example:

The 19th century saw a dramatic shift in the compositional conventions of orchestral music. As the traditional
understanding of tonality was challenged by brilliant musicians such as Beethoven and Mozart, lesser known
composers began experimenting with unorthodox modulations and aural textures. The fixation on contrapuntal
perfection gave way to more impressionistic soundscapes as composers strove less to make their work "right" and
more to make their music convey complex emotions.
Which word below is closest in meaning to "convey" as used in the passage above?
(A) transport
(B) communicate
(C) send
(D) grant
(E) hold

Choice B is correct. Choices A, C, and D are all synonyms for "convey," in isolation, but they are not appropriate in
the context of the passage above. Choice E is actually an antonymanother common type of wrong answer choice for
GMAT questions.

Consider the following passage:

I don't understand the necessity of following the news these days. News sources have proliferated, and many of these
sources publicize information that is distorted and extreme. What's more, they are notorious for leaving out crucial
bits of information. Vulgar television programs' focus on negative information appeals to viewers' desire for drama,
scandal, and spectacle.

In the above example, the author uses the word "spectacle" to highlight the dramatic and vulgar nature
of the TV programs.

Ivy-GMAT Exclusive GMAT-Prep for 99th percentile (760-800 range) aspirants not for everyone! Dont join us if you are
not willing to put in the requisite amount of work.
Consider the following passage:

Though many scholars expected the food industry to employ an increasingly artificial mode of production and
consumption after the discovery of new methods of preservation in the 1960s (when canned food became a standard
part of the American family diet), the next few decades actually saw a return to simple, organic foods, as well as an
emphasis on local farms and traditional methods of production. In the passage above, the author uses the word
"simple" to indicate
A. the coarse, unrefined nature of the food
B. the wholesome, natural quality of the food
C. the bland, uninteresting nature of the food
D. the artificial, popular nature of the food
E. the old-fashioned nature of the food

Choice B is correct. The passage states that the decades after the '60s saw a return to simple, organic foods, as
well as an emphasis on local forms and traditional methods of production. While the word "simple" could mean
"bland," "uninteresting," "coarse," or "unrefined" in another contexts, none of these accurately identify its role
within this sentence; here, "simple" means wholesome and natural. (Notice that, in the passage, "simple" is paired
with "organic" and "local.")

Locating and paraphrasing details in a passage

A detail is a specific statement that is explicitly denoted somewhere in a passage. In general, reading comprehension
questions test one's ability to absorb and process detailed information; part of this skill includes a knack for locating
details that might be buried within a passage.

In the majority of cases, the correct answer choice will be a "paraphrase" of a passage detail: a restatement of the
original fact in different words.

The rule of thumb for paraphrasing details is to make sure that the statement in the answer choice conveys the same
idea as the detail, but without introducing extreme words or ideas, distorting details, introducing topics that were not
mentioned in the passage, or substituting one term from the passage for another. In fact, correct details will often be
those that are a bit less specificrather than more specificas compared to the relevant text in the passage.

The correct answer to a detail question must have explicit support in the text of the passage. The correct answer will
either be a verbatim restatement or a paraphrase of relevant passage statements. Avoid the temptation to call upon
any outside knowledge about a topic; such knowledge could color one's interpretation of the answer choices,
sometimes steering the mind in the wrong direction. Remember, right answer choices always refer solely to
information in the text.

Passage Detail: One of Isaiah Berlin's most valuable and original contributions to political philosophy was his
differentiation of positive libertythe freedom to choose who governs societyfrom negative liberty, which he
defined as freedom from external interference.

Ivy-GMAT Exclusive GMAT-Prep for 99th percentile (760-800 range) aspirants not for everyone! Dont join us if you are
not willing to put in the requisite amount of work.
Paraphrase: Berlin's division of one political idea into two separate and distinct concepts was of great importance to
the field of political philosophy.

The GMAT increases difficulty on details questions in four distinct ways:

1. Often, details are buried within dense prose and written in convoluted or overly wordy ways.
2. The answer choices will commonly be statements paraphrased from the passage; they will almost never be
direct quotes. Some wrong answer choices will almost correctly paraphrase a detail, save for one or two
incorrect words.
3. The question stems themselves are sometimes confusing and wordy. A detail question may have a stem such
as, "According to the passage, which of the following is true about most technologies that undermine existing
business-to-business paradigm structures?"
4. Lastly, the GMAT will sometimes make detail questions trickier by asking for a detail that is NOT in the
passage. In these cases, four of the answer choices will provide details that can be found in the passage; since
finding and paraphrasing multiple details takes longer, questions of this type can be more time-consuming
(though not necessarily more difficult).

Consider the following passage:

Romanticism was a literary and artistic movement that encouraged individualism, glorified the ordinary, and often
included supernatural and mystical elements. Because Romantic poetry focused on the "beautiful" and the "sublime,"
two classically Romantic concepts spotted repeatedly throughout the works of poets such as William Blake, Lord
Byron, Samuel Coleridge, and John Keats, and was often set in gloomy, dramatic landscapes, Romanticism came, over
time, to be characterized by extreme, sentimental and overwrought emotions. The passage mentions all of the
following EXCEPT:
A. A definition of Romanticism that might explain its emphasis on the common man
B. A list of poets who embodied Romantic ideals
C. A sense of the setting one might find in a Romantic poem
D. A claim that Romantic poets engaged in excessive behavior
E. Several textual characteristics that surface in a number of Romantic works

Choice D is correct. The passage mentions that the "beautiful" and the "sublime" surfaced in the works of Blake,
Byron, Coleridge, and Keats and that their poetry involved extreme emotions and awe-inspiring landscapes. It does
not, however, mention anything about the personal lives of these poets.

Answers A, B, C, and E can all be found in the passage and are, therefore, incorrect. Choice A refers to passage
details that state that Romanticism encouraged individualism and glorified the ordinary, which might explain an
emphasis on the common man.

Consider the following passage:

A spectrum auction is a procedure whereby an organization uses an auction process to sell the rights to transmit
signals over specific electromagnetic wavelengths. Open to any group or individual that submits an application and an
upfront fee, spectrum auctions can last from a single day to several months. Since 1994, the Federal Communications
Commission has conducted auctions rather than awarding spectra through lotteries (or competitive hearings in which
the merits of each contender is considered by a jury). In order to reduce the time between application and grant, the
Commission decided to adopt the current auction process over its costlier and more bureaucratic alternatives.
According to the passage, the Federal Communications Commission elected to use the auction process
to license electromagnetic wavelengths because
(A) the public could be made beneficiary of the collected auction funds
(B) rights to specific electromagnetic wavelengths became available again in 1994
(C) competitive hearings were deemed more democratic than a lottery process
(D) it reduced the time between spectrum award and license grant
(E) it extended the decision-making process to a period that ranges from one day to several months

Choice D is correct. The auction process was adopted over alternatives such as the lottery and competitive hearing
processes because it was less bureaucratic, less expensive, and reduced the average time between application and
spectrum grant.

Ivy-GMAT Exclusive GMAT-Prep for 99th percentile (760-800 range) aspirants not for everyone! Dont join us if you are
not willing to put in the requisite amount of work.
Consider the following passage:

Though first synthesized in 1874, DDT's incredible potential as an insecticide was not fully understood until 1939. It
became an essential tool for the allied powers in World War II, who used it extensively for vector disease controlto
kill the insects that spread typhus and malaria, nearly eliminating these diseases in some areas.

It was only after WWII that DDT began to gain popularity as an agricultural pesticide. While a fair amount of
apprehension about its possible environmental hazards existed as early as the 1940s, these concerns would not rise to
the level of a national debate until twenty years later when early environmentalist Rachael Carson published her
highly influential book Silent Spring. The book claimed that DDT was a persistent toxinone that would remain in the
atmosphere for many years after its useand that it could cause several types of cancer in humans as well as
problems associated with decreased biodiversity and habitat destruction, particularly for birds. These findings
prompted an investigation by President Kennedy and eventually led to a nation-wide ban of the chemical in 1972.

Following the Stockholm Convention of 2004, DDT is now restricted from agricultural use but is still employed for
disease vector control in much of the world. Many epidemiologists have observed that the ban on agricultural DDT
has had the unexpected consequence of increasing its effectiveness for disease vector control. It has done so simply
by decreasing the overall volume of DDT in most environments. The most persistent obstacle facing those using DDT
for disease vector control is the development of resistance among target insect populations.
According to the passage, which of the following was an almost entirely overlooked consequence of
agricultural use of DDT?
(A) decreased biodiversity, particularly with regard to birds
(B) increased rates of several cancers in humans, particularly in agricultural communities
(C) decreased effectiveness of DDT as a disease vector control agent
(D) increased rates of malaria and other insect-borne diseases
(E) destruction of animal habitats essential to the overall ecosystem

Choice C is correct. While several of these other consequences are mentioned in the passage, the fact that
concerns about them were raised as early as the 1940s and then brought to national attention by Rachael Carson
argues that they were not, in fact, almost entirely overlooked. According to the passage, the fact that agricultural use
of DDT helped to increase resistance among mosquitoes and other insect populations (simply by exposing
environments to high levels of the chemical)therefore decreasing DDT's effectiveness as a disease vector control
agentwas not widely known until the institution of the 2004 ban.

Ivy-GMAT Exclusive GMAT-Prep for 99th percentile (760-800 range) aspirants not for everyone! Dont join us if you are
not willing to put in the requisite amount of work.
Consider the following passage:

The concept of "creative destruction" refers to the need for free economies to allow for the obliteration of older
companies in order to make way for new and more innovative companies that might be entering a market. There are
myriad examples available in modern society, from the abandonment of typewriters in favor of PC's, to the mass
conversion from Polaroid to digital photography. Though the term was coined by the German sociologist Werner
Sombart and later popularized by Austrian economist Joseph Schumpeter, many argue that it was actually the famed
English economic theorist Adam Smith who first identified the idea.

Pro-capitalist economists argue that Smith's concept of the "Invisible Hand" contains the original idea of creative
destruction. In his most famous work, The Wealth of Nations, Smith explicitly argued for the need for business to
failto get out of the way of newer technologies and more innovative producers. Though widely accepted in the
western world in the 19th century, Smith's ideas were strongly criticized by proponents of the 20th century economic
philosophy of John Keynes. According to Keynes' models, consumer spending can supersede creative destruction and
keep old producers in business; based on this model, politicians have been encouraged to use public money to prop
up failing industries for many decades.

Schumpeter's adherents counter the notion that propping up dying industries only delays the inevitable and slows
economic growth. They argue that it is wasteful to try to boost consumer spending with public subsidies, as this slows
down future growth by discouraging innovation and increasing tax liabilities. This is likely true, as studies of economic
growth rates throughout the 20th century suggest that economies grow faster when businesses are allowed to fail
naturally and when technological progress is unimpeded. The author would be most likely to agree with which
of the following statements regarding those who adhere to Keynesian economic models?
A. Their interest in supporting consumer spending by subsidizing faltering industries is misguided, as it only slows
down an unstoppable process.
B. They may underestimate the economic benefit of allowing certain industries to falter in the name of economic and
technological progress.
C. They are unreliable because consumer spending has no predictable relation to economic growth.
D. Their characterization of Smith's work is inaccurate, since Smith's concept of the "Invisible Hand" predates the
idea of creative destruction.
E. By overemphasizing the value of consumer spending, they overestimate the role that creative destruction plays in
stimulating growth

The key to answering this question correctly is to focus on the topic and the source: Keynesian economic models and
the author. What is the author's tone? The last paragraph gives us the answer: Here, the author agrees
with Schumpeter's adherents (who are critics of Keynes) in the sentence that begins "This is likely true..." However,
the author is not sharply critical of Keynes and does not necessarily disagree with his model on any points beyond this
specific detail.

Choice B is correct. One of the important aspects of this answer choice is its measured tone. It is not extreme like
answer choice C; it also does not distort passage details or confuse tones like answer choices A and B; answer choice
E, which is both an opposite answer and a distortion, is doubly wrong.

Ivy-GMAT Exclusive GMAT-Prep for 99th percentile (760-800 range) aspirants not for everyone! Dont join us if you are
not willing to put in the requisite amount of work.
Miscellaneous (rare) types of questions

Consider the following passage:

Mercury is only present in small amounts in marine ecosystems, yet it still presents a potential danger to fish eaters.
Species of fish that are relatively higher on the food chain contain higher concentrations of mercury in their tissues
than do smaller fish. Mercury is efficiently absorbed, but only very slowly excreted, by tiny organisms such as algae
and phytoplankton. When these small organisms are consumed by a predator, and those predators are consumed by
other predators, the mercury level accumulates. In order to reduce the natural occurrence of mercury
biomagnification, fisheries could study measures of plankton density, which are relatively easy and inexpensive to
estimate. Such measurements have been shown to be effective predictors of mercury biomagnification, wherein high
levels of density positively correlate with high levels of mercury concentration.
Many larger fishing companies, in order to assuage consumer concerns, have combined their marine fishing efforts
with farm fishing, which involves raising fish commercially in tanks or enclosures with no exposure to mercury or
other chemicals that are susceptible to biomagnification. Another advantage of fish farming and other forms of
aquaculture is that it counteracts the overfishing that has destroyed marine habitats in many ares while allowing
commercial fishermen to keep up with the increasing market demand for fish.
Which of the following, if true, most strongly supports the prediction that fisheries that measure marine
plankton density would reduce the amount of mercury in the fish they obtain from the ocean?
(A) Fish that live in areas of high plankton density are easier to obtain.
(B) Small-scale coastal fishermen have reported lower levels of mercury in fish obtained from areas of low plankton
density.
(C) High plankton density is also correlated with high mercury levels in fish farm tanks.
(D) Consumer concerns regarding mercury levels in fish have been greatly reduced due to the increase in fish
farming.
(E) Fishing companies that use farm fishing raise fish that live in tanks with high levels of plankton but have no
mercury in their tissues.

Choice B is correct. This prediction is supported by evidence that measures of plankton density have shown to be
effective predictors of mercury biomagnification. If, based on anecdotal statements, small-time fishermen have
observed a correlation between low mercury levels in fish and low plankton density, then the prediction that large
marine fisheries could use the same method to achieve lower levels of mercury in their fish is supportable.
Choices A, C, and E are probably true according to the passage, but none of these choices support the prediction.
Choice D is irrelevant.

Ivy-GMAT Exclusive GMAT-Prep for 99th percentile (760-800 range) aspirants not for everyone! Dont join us if you are
not willing to put in the requisite amount of work.
Consider the following passage:

The automotive industry is one of the most significant economic sectors in the world, and driving is the primary mode
of transportation in the majority of developed nations. While the automotive industry has brought innovation and
consistent improvement to transportation design, urban sprawl and automobile dependence have led to a number of
environmental problems. Cars and trucks are the largest users of petroleum, consuming almost half of what is
accrued each year, which is, in turn, the primary cause of greenhouse gas emissions. Dependence on crude oil is
largely due to the general publics over-reliance on automobiles.
Car companies are working on improving technologies that will reduce the amount of fossil fuel required by a car.
Hybrid cars combine a conventional internal combustion engine, which burns fossil fuel, with an electric propulsion
system, which is recharged by the motor itself. Other cars have placed signal lights on the dashboard to indicate to
motorists when they are driving within the optimum acceleration range for fuel economy. This way, the companies
reason, motorists can lower their own gas cost while simultaneously helping to reduce overall fuel consumption. The
growing use of these innovative automotive technologies is clearly responsible for a significant portion of the
reduction in greenhouse gases that has been observed over the last 10 years.

Which of the following, if true, would most weaken the author's claim that improvements in car
technology have contributed to reduced greenhouse gas emissions?
(A) Fewer hybrid and electric cars than standard cars have been sold in the last 10 years.
(B) The sale of cars and trucks that use abnormally large amounts of fuel has not decreased in the last 10 years.
(C) Greenhouse gas emissions caused by factories have not decreased in the last 10 years.
(D) Studies have shown that people have been driving less often, taking more public transportation, and carpooling
more often in the past 10 years.
(E) Studies have shown that people who drive hybrid cars drive twice as much as people who drive non-hybrid cars.

Choice D is correct. It presents an alternative cause, people driving less often, of the observed reduction in
greenhouse gas emissions, thus calling the author's conclusionthat innovative technology is causing this effectinto
question.

Ivy-GMAT Exclusive GMAT-Prep for 99th percentile (760-800 range) aspirants not for everyone! Dont join us if you are
not willing to put in the requisite amount of work.
Consider the following passage:

High taxes in one region lead to a gradual migration of residents into a region with lower taxes. This is because
capitalism incentivizes businesses to seek the highest possible return on investments, and it generally costs
businesses less to work in regions with lower taxes. Low business costs make it easier for companies to increase their
returns, which translates to more investment, growth, and, ultimately, more jobs in the lower tax region.

This excerpt is pure abstraction; no specific town or state is cited, nor is any specific business, time period, or
instance. A specific case that exemplifies this idea might read something like this:

Census data over the last data shows a clear migration trend within the United States; people are moving away from
coastal states, including New York and California. Both states have relatively high income, property, and sales tax
rates. By contrast, Texas, a state with historically low taxes, has seen an influx of over millions of new residents.
We can infer the second paragraph using the first.

In general, political parties are complicated amalgamations of various interest groups, each vying for
power. As a result, political party platforms tend to change very slowly, as each faction clings to power
in spite of mounting public opposition. One example of this generalization would be ________
(A) the century-long struggle to attain equal voting and education rights for black Americans after the Civil War.
(B) the fact that the two-party system remains firmly in place, in spite of a recent poll that shows that Americans
support the formation of more political parties.
(C) the conservative party's long and difficult conversion to support for more liberal divorce laws, much to the chagrin
of the fringe religious groups that long supported them.
(D) the electoral college system, which awards an entire state's votes to the winner of the popular vote in a
presidential election, regardless of the proportion of voters for the other candidate.
(E) the conflicting interests of health-care reform advocates and trial attorneys, both groups that are pillars of the
liberal party.

Choice C is correct. Remember that the principle we are trying to make manifest is the slow change of political
parties due to factionalization. Our concern is neither with interest group warfare (E) nor with slow change (A) in and
of themselves. Choice B is out of scope, and choice D is irrelevant to the argument.

Ivy-GMAT Exclusive GMAT-Prep for 99th percentile (760-800 range) aspirants not for everyone! Dont join us if you are
not willing to put in the requisite amount of work.
Consider the following passage:

Maintaining a happy and hard-working workforce is one of the core challenges for business managers in todays
competitive economy. To promote greater flexibility, individual accountability, and satisfaction with work, many
human resource managers have begun introducing flex time systems in favor of traditional work hours. Unlike the
conventional work day, wherein employees are required to arrive at work and leave at specific times (9 am to 5 pm,
traditionally), the flex time system allows workers to arrive and leave times of their choosing. As long as employees
work a set number of hours each week or complete a set number of tasks, they are free to choose when they are at
the office.

Research has shown that flex time, while somewhat trendy and popular with younger office managers, is not
appropriate for all office environments. Service industries, such as travel agencies and consulting firms, have been
found to suffer losses in efficiency and profitability after adopting flex time systems. Though it is not entirely clear
why this is so, many have speculated that this may be a result of the emphasized need for one-one-one human
interaction, or face time, in these fields. On the other hand, computer programmers, systems analysts, and other
employees capable of doing the bulk of their work through telecommunication, have been found to become more
efficient and content with their work in flex time systems. ________________________________________

Which of the following would provide the most logical continuation of the final paragraph of the
passage?
(A) We can conclude that all other types of workers would likely see a decrease in efficiency as a result of the
adoption of a flex time system.
(B) As a result, most service sector industries do not allow their tech workers to participate in a flex time system.
(C) This is because technology specialists, unlike most other employees, prefer to work a set, predictable schedule
with the same start and stop time each day.
(D) Therefore, technology companies would likely benefit from the adoption of flex time systems for their employees.
(E) Increases in salary and benefits have also been shown to motivate technology workers and improve their work
satisfaction.

Choice D is correct.

This passage presents a good opportunity to examine the contrast between a conclusion and a concluding
sentence. Take a moment to consider appropriate conclusions and concluding sentences (for the 2nd paragraph),
respectively:

The passage itself is about flex time, how it works, and its effects on a few different types of companies. The
conclusion is basically that flex time works for some types of workers, but not for others.

The concluding sentence, however, has to consider the specific context of the sentence before it. That last sentence
states that workers who can function through telecommunication (tech workers are listed as examples) see efficiency
gains and increased work satisfaction. Therefore, answer choice D is the most logical continuation, as it recommends
flex time for precisely these workers.

Ivy-GMAT Exclusive GMAT-Prep for 99th percentile (760-800 range) aspirants not for everyone! Dont join us if you are
not willing to put in the requisite amount of work.
Consider the following passage:

In June 2004, Hirudo medicinalis, blood-sucking leeches, became the second living organisms permitted by the Food
and Drug Administration to be used as a medical devices; the first organisms were maggots. Mainstream medicine
uses leeches sparingly, generally in microsurgery and plastic surgery. In these cases, the leech relieves venous
congestion, or the over-accumulation of blood in veins due to obstruction of flow. Such congestion, which results in
harmful blood-clotting, can plug arteries that carry nourishment to body tissuesnourishment without which these
tissues will die.
In medieval and early modern medicine, medicinal leeches were used to remove blood from patients as part of a
process meant to balance the "humors" and aid in the proper functioning of the human body. The four humors were
said to be blood, phlegm, black bile, and yellow bile. Currently, leeches are used for their ability to attach to their
hosts and feed off their blood. Leeches are especially well-suited to relieve venous congestion, not only because they
sucks out excess blood, but also because their saliva contains an anti-clotting enzyme, hirudin, that prevents the
blood from accumulating. This anti-clotting enzyme helps the leech feed on its host's blood and aids doctors in
promoting blood flow in patients experiencing a blockage.
Surgeons recently used leeches to help re-attach the hand of a surfer who had been attacked by a great white shark.
As doctors were working to repair the damage to the victims wrist, they noticed a sluggishness in the surfer's blood
circulation. They eventually decided to attach the blood-sucking creatures to the patient's arm, an operation which
resulted in the recovery of the surfer's hand.
Which of the following titles best summarizes the contents of the passage?
(A) Hirudo medicinalis: blood sucking Leeches
(B) Medical uses of Hirudo medicinalis
(C) Drawbacks to the use of Hirudo medicinalis in modern medicine
(D) Hirudo medicinalis: A primary example of a return to medieval medicine
(E) Advantages to the use of Hirudo medicinalis in microsurgery

Choice B is correct. It is the only title that encompasses the entire passage and does not refer to anything outside
of its scope.

Consider the following passage:

Companies that invest heavily in the expansion of their own infrastructure sometimes face an unexpected dilemma,
especially when they are operating within a limited geographical area; multiple arms of the same business often
attract the same customer base, thereby saturating a market. Not only does the opening of new divisions increase
company costs, but it also creates a situation wherein divisions (within the same company) are in competition with
one another. This, in turn, raises the company's overhead costs without increasing its revenue.
Which of the following situations best exemplifies the "dilemma" discussed in the passage?
(A) An electronics store opens new branches in new regions as profits increase.
(B) A bookstore opens a video rental business to appeal to customers who do not read books.
(C) A print advertising company opens an elegant new office that provides web marketing services.
(D) A local bakery opens a nearby coffee shop that offers coffee and some baked goods.
(E) A large fast food chain begins to offer delivery services to expand its customer base.

In this example, the principle is far more specific; the right answer will reflect a company that operates in a limited
geographical area and that has expanded, thus creating new divisions and fomenting unprofitable competition within
the company. Choices A ("new regions"), B (no competition), C (different service) and E (no new competition) can all
be eliminated. Only choice D meets the necessary criteria. Choice D is correct.

Ivy-GMAT Exclusive GMAT-Prep for 99th percentile (760-800 range) aspirants not for everyone! Dont join us if you are
not willing to put in the requisite amount of work.
Consider the following passage:

The workers' union is right to believe that lay-offs at FlexClick will ultimately diminish the strength of the business; job
loss at FlexClick will negatively impact the local economy and will negate the company's efforts to revitalize the area.
According to FlexClick, the layoffs will result in a simultaneous streamlining of operations and a doubling of profit
margins. Executives at FlexClick are naive to believe they will be able to cut a good percentage of their staff and yet
maintain their excellent public relations (and continue to attract business from clients who believe they are supporting
local workers by supporting FlexClick). However, job loss is inevitable in this region, whether the union agrees with
the company's decisions or not.

According to the passage, the workers' union and the executives at FlexClick disagree about which of
the following points?
A. the inevitability of job loss in the region concerned
B. whether the loss of jobs at FlexClick will ultimately diminish the strength of the business
C. whether it is possible for FlexClick to streamline operations and decrease profit margins at the same time
D. whether FlexClick needs to maintain excellent public relations
E. whether the company's efforts to double profit margins is incompatible with its public image

Choice B is correct. The workers' union and the executives at FlexClick disagree about whether the loss of jobs at
FlexClick will ultimately diminish the strength of the business. The workers' union believes that FlexClick's plan to lay
off employees, streamline operations, and double profit margins will ultimately detract from the business, as it will
tarnish FlexClick's ability to attract clients who believe they are supporting local workers by supporting FlexClick. To
answer this question correctly, we need to identify the tones of the author, the workers' union, and the FlexClick
executives and carefully note which tones the question stem is asking us to compare.

Consider the following passage:

The fact that lowering product price can create an advantage for one company in a competitive market does not
indicate that every product, if set at a lower price, will sell more effectively. The demand for a product, as well as the
quality of that product, must be considered when one is developing a marketing plan and setting a product's price
point. If a product's quality is similar to that of others in the same field, and the demand is stable, then lowering the
price too much will simply reduce revenue without significantly increasing sales. In cases of similar quality, a company
simply needs to offer its product at a price slightly lower than that of its competitorsand no lower than thatin
order to achieve the desired competitive advantage.
This fact was not apparent to one industrial metals manufacturer, who failed to stay competitive with
others in the field after these competitors lowered their material prices below a certain point. What the
industrial metals manufacturer did not realize was that the dramatic price drop instituted by the competitors sent
customers an implicit message that the competitors' metals were of lower quality than other metals on the market.
Were the industrial metals manufacturer to realize that it needed only to wait for the return of customers who value
quality, this manufacturer would not have seen the need to engage in a pricing war.
The decrease in prices, however, was not evaluated in its effect on customer perception of value.

The author includes the highlighted statement in the second paragraph in order to
(A) describe an exceptional case in which the argument does not apply
(B) undermine a position by illustrating the pitfalls of employing a particular strategy
(C) offer a hypothetical situation to which the author's argument applies
(D) refute an argument put forth in the first paragraph
(E) introduce an example that illustrates the truth of an assertion outlined in the first paragraph

Choice E is correct. The actions of this metals manufacturer serve as an example of the point made by the author in
the first paragraphthat lowering the price of a product will not always lead to a competitive market advantage. The
phrase this fact [that lower price does not consistently lead to market advantage] was not apparent to one... shows
that the author is about to introduce an "example" that illustrates the "assertion" in the first sentence. Not that
answer choices beginning with the words "undermine" and "refute" can be eliminated almost immediately. Choice C is
incorrect because we have reason to believe that the situation with the industrial metals manufacturer was real, not
"hypothetical."
Consider the following passage:

Ivy-GMAT Exclusive GMAT-Prep for 99th percentile (760-800 range) aspirants not for everyone! Dont join us if you are
not willing to put in the requisite amount of work.
The use of antidepressant medications in treating patients with depression has grown exponentially since the
manufacturers of these prescription drugs began advertising their products on television. A study completed by Dr.
Rosen, a psychologist at the University of XYZ, concludes that direct-to-consumer advertising of prescription drugs
serves to increase pharmaceutical sales and, unfortunately, leads to potential overuse. Dr. Rosen emphasizes that this
is particularly true of mood-altering medications for illnesses that could be better treated with cognitive
psychotherapy. Patients turn to medications to treat their symptoms without seeking the therapy they need and, as a
result, are experiencing negative side effects which, in extreme cases, can lead to suicide. Dr. Rosen suggests that
the one positive outcome of television advertisements for prescription medications is they have increased public
awareness of the seriousness and prevalence of some of the diseases treated by these medications.
The author uses the phrase "the one positive outcome" at the end of the passage in order to:
(A) highlight the peculiarity of the pharmaceutical companies' advertising strategy
(B) emphasize the low value of prescription medication television advertisements
(C) distinguish the positive outcomes of medication advertisements from the positive outcomes of psychotherapy
(D) single out one example of medication advertisements leading a patient to recovery
(E) deny the claim that medication advertisements are useless

Choice B is correct. The phrase "one positive outcome" is used by the author to highlight the fact that there
are many negative outcomes. All the other outcomes of television advertisements of prescription medications
mentioned in the passage are negative.

Ivy-GMAT Exclusive GMAT-Prep for 99th percentile (760-800 range) aspirants not for everyone! Dont join us if you are
not willing to put in the requisite amount of work.

You might also like